3

Find $\dfrac{dy}{dx}$ if $y=\sin^{-1}\bigg[\dfrac{5\sin x+4\cos x}{\sqrt{41}}\bigg]$

My Attempt

Put $\cos\theta=5/\sqrt{41}\implies\sin\theta=4/\sqrt{41}$ $$ y=\sin^{-1}\big[\sin(x+\theta)\big]\implies\sin y=\sin(x+\theta)\\ y=n\pi+(-1)^n(x+\theta)\\ \boxed{\frac{dy}{dx}=(-1)^n} $$

But my reference gives the solution $y'=1$, am I missing something here ?

Quanto
  • 97,352
Sooraj S
  • 7,573
  • 1
    Your solution is incorrect because of the $n\pi$. $y$ from its definition can only be in the range of values $\Bigr[-\frac{\pi}{2},\frac{\pi}{2}\Bigr]$ because of the range of arcsin – Ninad Munshi Nov 29 '19 at 19:21
  • @NinadMunshi But the sin function can have any domain. So $x+\theta$ need not have to be in the range $[-\pi/2,\pi/2]$ – Sooraj S Nov 29 '19 at 19:26
  • @NinadMunshi $y=\sin^{-1}\big[\sin(x+\theta)\big]=x+\theta$ iff $x+\theta\in[-\pi/2,\pi/2]$, right? but it is not specified, thats why I had to follow the remaining steps. – Sooraj S Nov 29 '19 at 19:32
  • 1
    See https://math.stackexchange.com/questions/672575/proof-for-the-formula-of-sum-of-arcsine-functions-arcsin-x-arcsin-y – lab bhattacharjee Nov 30 '19 at 02:27

3 Answers3

4

As you correctly pointed out we have

$$y = \arcsin\left[\sin\left(x +\theta\right) \right],$$ where $\theta = \arcsin\frac4{\sqrt{41}}$.

Now observe that ($k\in \Bbb Z$) $$ \arcsin\sin \alpha = \begin{cases}\alpha-2k\pi & \left(2k\pi-\frac{\pi}2\leq \alpha < 2k\pi+\frac{\pi}2\right)\\ - \alpha - (2k-1)\pi & \left(2k\pi+\frac{\pi}2\leq \alpha < 2k\pi+\frac{3\pi}2\right).\end{cases} $$

Thus your function is the triangular wave (see Figure below)

$$ y = \begin{cases} x +\theta- 2k \pi & \left(2k\pi-\frac{\pi}2-\theta\leq x < 2k\pi+\frac{\pi}2-\theta\right)\\ -x-\theta-(2k-1)\pi-\theta & \left(2k\pi+\frac{\pi}2-\theta\leq x< 2k\pi+\frac{3\pi}2-\theta\right) \end{cases} $$

whose derivative is

$$ y = \begin{cases} 1 & \left(2k\pi-\frac{\pi}2-\theta< x < 2k\pi+\frac{\pi}2-\theta\right)\\ -1 & \left(2k\pi+\frac{\pi}2-\theta< x<2 k\pi+\frac{3\pi}2-\theta\right) \end{cases} $$ enter image description here

dfnu
  • 7,528
0

I think your reference is wrong.

$y'=1$ is true half the time - the other half $y'=-1$ as you wrote.

marty cohen
  • 107,799
0

Rewriting,

$$\sin y=\dfrac{5\sin x+4\cos x}{\sqrt{41}}=\sin(x+\theta)\tag 1$$

with $\theta=\tan^{-1}\frac4{5}$ and recognize $y\in[-\frac\pi2,\frac\pi2]$, hence $\cos y \ge 0$,

$$\cos y= \text{sgn}[\cos(x+\theta) ]\cos(x+\theta) $$

Take the derivative of (1),

$$\> y' = \dfrac{\cos(x+\theta)}{\cos y} = \dfrac{\cos(x+\theta)}{\text{sgn}[\cos(x+\theta) ]\cos(x+\theta)} \\ =\text{sgn}[\cos(x+\theta) ]= \begin{cases} 1, & x+\theta -2k\pi \in \left(-\frac{\pi}2,\frac{\pi}2\right]\\ -1, & x+\theta -2k\pi\in \left(\frac{\pi}2,\frac{3\pi}2\right] \end{cases} $$

which, as seen in the graph below, alternates at either -1 or 1 depending on the value of $x$.

enter image description here

Quanto
  • 97,352